Bonjour,
On ne peut évidemment pas être sûr, mais ...
Je parierais qu''il y a une erreur de recopie d'énoncé, le z=4-√3i est suspect (même si c'est évidemment possible).
Bonjour,
Cet exercice est un classique.
On le trouve un peu partout sur le web, en 2004, 2008, 2014,...
Par exemple ici , il y a 3 ans :
https://math.stackexchange.com/questions/3796196/how-can-i-resolve-this-equation-z4-2z3-7z2-−-18z-26-0-where-there
L'énoncé doit demander de calculer P(1+i)P(1+i)P(1+i)
On trouver P(1+i)=0P(1+i)=0P(1+i)=0
1+i\boxed{1+i}1+i est donc solution de l'équation P(z)=0P(z)=0P(z)=0
La première question traitée permet de déduire que 1−i\boxed{1-i}1−i est aussi solution de l'équation P(z)=0P(z)=0P(z)=0
P(z)=0P(z)=0P(z)=0 peut donc se transformer en : (z−(1+i))(z−(1−i))(az2+bz+c)=0(z-(1+i))(z-(1-i))(az^2+bz+c)=0(z−(1+i))(z−(1−i))(az2+bz+c)=0
Par exemple par identification, on peut trouver les valeurs de a,b,ca,b,ca,b,c
Après calculs a=1,b=−4,c=13a=1,b=-4,c=13a=1,b=−4,c=13
On résout l'équation du second degré z2−4z+13=0z^2-4z+13=0z2−4z+13=0
Après calculs; Δ=−36=(6i)2\Delta=-36=(6i)^2Δ=−36=(6i)2
Deux solutions 2+3i\boxed{2+3i}2+3i et 2−3i\boxed{2-3i}2−3i
On a obtienu ainsi les 4 solutions de l'équation :
z4−6z3+23z2−34z+36=0\boxed{z^4-6z^3+23z^2-34z+36=0}z4−6z3+23z2−34z+36=0
Bons calculs à tous ceux qui souhaitent les faire .
@Zeïnab-Mahamadou , l'énoncé indique qu'il faut que tu justifies que la fonction fff est définie sur l’ensemble UUU des nombres complexes de module 1
Tu dois donc prouver que pour tout zzz de module 1 (c'est à dire ∣z∣=1|z|=1∣z∣=1) , f(z)f(z)f(z) est calculable.
Bonjour,
Peut être à coté de ce qui est attendu.
Exponentielle sur C comme le titre du topic l'indique.
e(x+i.y)=ex∗ei.ye^{(x + i.y)} = e^x * e^{i.y}e(x+i.y)=ex∗ei.y
e(x+i.y)=ex∗(cos(y)+i.sin(y))e^{(x + i.y)} = e^x * (cos(y) + i.sin(y))e(x+i.y)=ex∗(cos(y)+i.sin(y))
Pour que e(x+i.y)=0e^{(x + i.y)} = 0e(x+i.y)=0 il faudrait que :
a) soit que exe^xex soit nul (ce qui est impossible (à montrer par Taylor ou autrement))
b) soit que cos(y)+i.sin(y)=0cos(y) + i.sin(y) = 0cos(y)+i.sin(y)=0 ce qui est impossible car il n'existe pas de réel y qui annule à la fois cos(y) et sin(y) (puisque cos²(y) + sin²(y) = 1 et donc ...)
@vittoriaisadora , bonjour,
Pour la question 1 ), tu as dû trouver :
(1+i)(1+2i)(1+3i)=−10(1+i)(1+2i)(1+3i)=-10(1+i)(1+2i)(1+3i)=−10
Pistes à expliciter avec rigueur pour la 2) (il n'y a que les idées, j'ai fait vite...)
L'argument principal de −10-10−10 est π\piπ vu que −10-10−10 est un réel strictement négatif.
arg[(1+i)(1+2i)(1+3i)]=πarg[(1+i)(1+2i)(1+3i)]=\piarg[(1+i)(1+2i)(1+3i)]=π
arg(1+i)+arg(1+2i)+arg(1+3i)=π\boxed{arg(1+i)+arg(1+2i)+arg(1+3i)=\pi}arg(1+i)+arg(1+2i)+arg(1+3i)=π
Cercle trigonométrique :
Pour tout θ\thetaθ compris entre −π2-\dfrac{\pi}{2}−2π et +π2+\dfrac{\pi}{2}+2π
arg(1+iθ)=Arctanθarg(1+i\theta)=Arctan\thetaarg(1+iθ)=Arctanθ
Tu prends θ=1\theta=1θ=1 , θ=2\theta=2θ=2 et θ=3\theta=3θ=3 et tu dois trouver la propriété demandée.
@srhmrc a dit dans Forme trigonométrique :
@Black-Jack Merci! Est-ce correct d'écrire 21°,8014 ? Ou doit-on plutôt écrire 21,8014° ?
Moi, je préfère 21,8014°, mais les conventions d'écriture ne sont malheureusement pas les mêmes pour tous.
Bonjour,
Je montre le point a.
f(i.b) = (i.b)^4 - 10.(i.b)³ + 38.(i.b)² - 90.(ib) + 261
f(ib) = b^4 + 10i.b³ - 38b² - 90i.b + 261
f(i.b) = (b^4 - 38b² + 261) + i.(10.b³ - 90.b)
f(i.b) = 0 ---> le système :
b^4 - 38b² + 261 = 0 (1)
10.b³ - 90.b = 0 (2)
(2) --> 10.b.(b²-9) = 0
10.b.(b-3).(b+3) = 0 --> b = 0 OU b = -3 ou b = 3
b = 0 remis dans (1) donne 261 = 0 (et donc b = 0 ne convient pas)
b = -3 ou b = 3 remis dans (1) donne : 3^4 - 38*3² + 261 = 81 - 342 + 261 = 0
Donc b = -3 et b = 3 satisfont le système (1) et (2)
Conclusion : z = -3i et z = 3i sont solutions de f(z) = 0
'''''''''
A toi pour le b.
@noamii Bonsoir,
Un point MMM est dit invariant par l’application fff (transformation) si, et seulement si, f(M)=Mf (M) = Mf(M)=M.
Pour un nombre complexe zzz, pour déterminer l’ensemble des points invariants par l’application fff , résoudre l’équation f(z)=zf (z) = zf(z)=z et déduire l’ensemble des points vérifiant cette égalité (droite, cercle, etc...).
C'est très bien @Rémy-DC .
Je pense que la fin de ton exercice te demandait de résoudre l'équation P(z)=0P(z)=0P(z)=0 et que tu l'as résolue sans difficulté.
Bon travail !
@floflowww ,
Autre façon sans résoudre l'équation,
Soit aaa une solution réelle éventuelle :
a3+(1+2i)a2−3(1+i)a−2+2i=0a^3+(1+2i)a^2-3(1+i)a-2+2i=0a3+(1+2i)a2−3(1+i)a−2+2i=0
en mettant le membre de gauche sous forme algébrique :
(a3+a2−3a−2)+i(−2a2−3a+2)=0(a^3+a^2-3a-2)+i(-2a^2-3a+2)=0(a3+a2−3a−2)+i(−2a2−3a+2)=0
D'où le système :
{a3+a2−3a−2=0−2a2−3a+2=0\begin{cases}a^3+a^2-3a-2=0\cr -2a^2-3a+2=0\end{cases}{a3+a2−3a−2=0−2a2−3a+2=0
En résolvant dans RRR l'équation −2a2−3a+2=0-2a^2-3a+2=0−2a2−3a+2=0 , tu dois trouver, sauf erreur :
a=−2a=-2a=−2 ou a=12a=\dfrac{1}{2}a=21
En substituant dans la première équation, tu dois trouver que a=−2a=-2a=−2 convient et que a=12a=\dfrac{1}{2}a=21 ne convient pas.
Conclusion :
−2-2−2 est donc la seule solution réelle de l'équation.
Je pense que c'est plutôt cette méthode qui est attendue dans ton DM.
A toi de voir, bien sûr.
Une fois de plus, le site supprime certains caractères des messages ...
J'essaie de les rajouter ici, ma fin de message devrait être :
(f(z)-z)/(1+2i) = 1/30 * ((5+10i)z + (5 - 10i)z\ ) (1)
Or (1-2i)(-3+4i) = -3+4i + 6i + 8 = 5+10i
Et donc (1) --> (f(z)-z)/(1+2i) = 1/30 * ((1-2i)(-3+4i) z + (1-2i)*5z\ )
(f(z)-z)/(1+2i) = 1/30 * (1-2i)*[(-3+4i) z + 5z\ ]
**Pour les gestionnaires du site :
Serait-il possible de corriger ce bug (caractères supprimés par le site dans les messages) ?
Dans le cas présent, certains caractères " \ " ont été supprimés.
Si il sont suivi par un "espace" ou une lettre, ils sont bien transcrits, mais si ils sont suivi par un signe de ponctuation ou une parenthèse ou un crochet ou ... , alors le site ne les affiche pas.**
Bonjour une partie de mon dm me pose problème : il faut calculer la norme du vecteur MnMn+1 en fonction de n sachant que l'affixe du point Mn est Zn = rn(cosθn +i sinθn)
Avec rn = 3*(2/3)^n et θn = 0+(2pi/3)*n
J'ai donc écrit : IIMnMn+1II = Izn+1 - znI
= I rn+1e^i(θn+1) -(rne^i(θn))I
mais après je ne vois pas comment calculer cette expression en fonction de n
Bonjour,
@Junior-KMD , @Noemi t'a déjà mis sur la voie.
Si tu as besoin, tu peux consulter ici (exercice 1)
https://studylibfr.com/doc/1892397/ds-n˚1---nombres-complexes---correction
Reposte si besoin.
@MS34 Bonsoir,
Le scan de l'énoncé de l'exercice est interdit sur ce forum. Seuls les scans de schémas, figures ou graphiques sont autorisés.
Ecris l'énoncé, indique tes éléments de réponse et la question qui te pose problème et tu obtiendras des pistes de résolution.
Le scan va être supprimé.
Parfait @Livindiam-Livin si tu as compris.
Tu peux déduire que pour tout n de NNN, zn=z0+nrz_n=z_0+nrzn=z0+nr , c'est à dire zn=3+i+n(2−i)z_n=3+i+n(2-i)zn=3+i+n(2−i)
Bon travail.
@mathsandbooks , si par hasard, la première indication était la bonne Soit N un point quelconque sur la droite passant par B, parallèle à l'axe des imaginaires purs ", en utilisant l'expression de zBz_BzB que tu as choisi: zB=5−iz_B=5-izB=5−i, tu peux exprimer zNz_NzN : zN=5+iyz_N=5+iyzN=5+iy
En calculant zN′z_{N'}zN′, avec tes valeurs, sauf erreur, tu dois trouver zN′=y+23+3iz_{N'}=\dfrac{y+2}{3}+3izN′=3y+2+3i
Lorsque y varie de −∞-\infty−∞ à +∞\infty∞, y+23\dfrac{y+2}{3}3y+2 varie aussi de −∞-\infty−∞ à +∞\infty∞, donc le lieu de N′N'N′ est la droite parallèle à l'axe des réels passant par le point d'affixe 3i.
@Luukao-_ , l'expression que tu iindiques n'est pas claire...
S'agit-il de f(z)=z−3+iz+5−3if(z)=\dfrac{z-3+i}{z+5-3i}f(z)=z+5−3iz−3+i ?
Si c'est ça, il fallait mettre des parenthèses pour séparer le numénateur du dénominateur (si tu n'utilises pas le Latex)
Avec cette expression :
f(zˉ)=zˉ−3+izˉ+5−3if(\bar z)=\dfrac{\bar z-3+i}{\bar z+5-3i}f(zˉ)=zˉ+5−3izˉ−3+i
En appliquant les propriétés des conjugués :
f(zˉ)‾=zˉ−3+i‾zˉ+5−3i‾\overline{f(\bar z)}=\dfrac{\overline{\bar z-3+i}}{\overline{\bar z+5-3i}}f(zˉ)=zˉ+5−3izˉ−3+i
f(zˉ)‾=z−3−iz+5+3i\overline{f(\bar z)}=\dfrac{ z-3-i}{z+5+3i}f(zˉ)=z+5+3iz−3−i
Si ce n'est pas de cette expression dont il s'agit, applique la même démarche avec une autre expression.
@yasmi16 , oui la valeur de jjj que tu donnes est bonne.
C'est la notation usuelle de jjj en mathématiques (je précise), car dans certaines autres disciplines jjj est utilisé autrement ( à la place de iii).
j=e2iπ3=−12+i32j=e^{\dfrac{2i\pi}{3}}=-\dfrac{1}{2}+i\dfrac{\sqrt 3}{2}j=e32iπ=−21+i23
Regarde le lien ici
https://fr.wikipedia.org/wiki/J_(nombre_complexe)
Mais , si tu fais toutes les vérifications des 4 conditions, dans cet exercice, tu peux aussi donner à jjj la valeur
e4iπ3=−12−i32e^{\dfrac{4i\pi}{3}}=-\dfrac{1}{2}-i\dfrac{\sqrt 3}{2}e34iπ=−21−i23
Ces deux valeurs sont des valeurs conjuguées.
Dans ton exercice, tu dois donner les deux valeurs possibles.
Remarque : Si ton énoncé avait précisé, en plus, en 5ième condition, que la partie imaginaire de jjj était strictement positive ( ce qui aurait été heureux, je pense...) tu n'aurais trouvé que j=e2iπ3=−12+i32j=e^{\dfrac{2i\pi}{3}}=-\dfrac{1}{2}+\dfrac{i\sqrt 3}{2}j=e32iπ=−21+2i3, qui est la valeur usuelle.
@thomas15465 , bonsoir,
Politesse à ne pas oublier.
Pour prouver que le quadrilatère est un parallélograme, tu peux, par exemple, calculer l'affixe de AB→\overrightarrow{AB}AB qui vaut zB−zAz_B-z_AzB−zA et l'affixe de DC→\overrightarrow{DC}DC qui vaut zC−zDz_C-z_DzC−zD (et tu dois trouver pareil)
Bonjour,
zI=(−2+i)+(5)2z_{I} = \frac{(-2+i)+(5)}{2} zI=2(−2+i)+(5)
zI=32+12.iz_{I} = \frac{3}{2} + \frac{1}{2}.i zI=23+21.i
Si tu as compris ... tu pourras déterminer l'affixe de J
A partir de z = 1/(3+4i)
On multiplie numérateur et dénominateur par (3 - 4i)
--> z = (3 - 4i) /[(3-4i)(3+4i)]
Or (3-4i)(3+4i) = 25 et donc ...
z = (3-4i)/25
z = (3/25) - (4/25).i
De rien @nathan0202 ,
Si tu as bien compris la méthode, tu peux maintenant traiter le cas de 1−i1-i1−i
1−i=2 e−iπ41-i=\sqrt 2\ e^{-i\dfrac{\pi}{4}}1−i=2 e−i4π
Tu dois trouver que les racines 4ème de (1-i) s'écrivent :
z=28 ei(−π16+2kπ4)\boxed{z=\sqrt[8]2\ e^{i(\dfrac{-\pi}{16}+\dfrac{2k\pi}{4})}}z=82 ei(16−π+42kπ) pour k∈Zk\in Zk∈Z
Tu peux bien sûr écrire : z=28 ei(−π16+kπ2)\boxed{z=\sqrt[8]2\ e^{i(\dfrac{-\pi}{16}+\dfrac{k\pi}{2})}}z=82 ei(16−π+2kπ) pour k∈Zk\in Zk∈Z
Ces racines ont pour images les points E,F,G,H du schéma ci dessous
Bons calculs.
Conséquence : on peut mettre (z−2i)(z-2i)(z−2i) en facteur.
P(z)=(z−2i)(az2+bz+c)P(z)=(z-2i)(az^2+bz+c)P(z)=(z−2i)(az2+bz+c)
On précède par identification avec l'espression de départ, pour trouvera,b,ca,b,ca,b,c.
Après calculs : a=1,b=1,c=−1+3ia=1, b=1, c=-1+3ia=1,b=1,c=−1+3i
Donc :
P(z)=(z−2i)(z2+z−1+3i)\boxed{P(z)=(z-2i)(z^2+z-1+3i)}P(z)=(z−2i)(z2+z−1+3i)
P(z)=0P(z)=0P(z)=0 <=>z=2iz=2iz=2i ou z2+z−1+3i=0z^2+z-1+3i=0z2+z−1+3i=0
z2+z−1+3i=0z^2+z-1+3i=0z2+z−1+3i=0 équation du second degré.
Δ=5−12i\Delta=5-12iΔ=5−12i
Il faut chercher les racines carrées complexes +δ+\delta+δ et −δ-\delta−δ de 5−12i5-12i5−12i :
(α+iβ)2=5−12i(\alpha+i\beta)^2=5-12i(α+iβ)2=5−12i
Après développements et regroupements , et ajoutant l'égalité des modules poursimplifier la résolution, on obtient :
{α2−β2=5αβ=−6α2+β2=13\begin{cases}\alpha^2-\beta^2=5 \cr \alpha\beta=-6\cr \alpha^2+\beta^2=13 \end{cases}⎩⎪⎪⎨⎪⎪⎧α2−β2=5αβ=−6α2+β2=13
D'où :
les racines carrées complexes de 5-12i$ sont +δ=3−2i+\delta=3-2i+δ=3−2i et −δ=−3+2i-\delta=-3+2i−δ=−3+2i
Les solutions de z2+z−1+3i=0z^2+z-1+3i=0z2+z−1+3i=0 sont , après utilisation des formules usuelles,
z1=−1+3−2i2=1−iz_1=\dfrac{-1+3-2i}{2}=1-iz1=2−1+3−2i=1−i et z2=−1−3+2i2=−2+iz_2=\dfrac{-1-3+2i}{2}=-2+iz2=2−1−3+2i=−2+i
Conclusion :
Les solutions de P(z)=0P(z)=0P(z)=0 sont 2i,1−i,−2+i\boxed{2i,1-i,-2+i}2i,1−i,−2+i
Bon courage pour les calculs !
@Joyca2 ,
Oui pour le cercle.
Ta simplification pour l'équation de la droite est fausse.
3y−3−x=03y-3-x=03y−3−x=0 <=> 3y=x+33y=x+33y=x+3 <=> y=13x+1y=\dfrac{1}{3}x+1y=31x+1
Bonjour,
Comme au départ, j'avais suggéré à @Joyca2 de chercher les racines 4ème de 1, je me sens un peu obligée de terminer les pistes relatives à cette voie...
Donc, déjà trouvé, Z4=1Z^4=1Z4=1 a pour solutions 1,−1,i,−i1,-1, i,-i1,−1,i,−i
Retour à zzz, avec la condition z≠1z\ne 1z=1
1er cas :2z+1z−1=1\dfrac{2z+1}{z-1}=1z−12z+1=1 c'est à dire 2z+1=z−12z+1=z-12z+1=z−1
On transpose
Equation du premier degré ; solution z=−2z=-2z=−2
2ème cas :2z+1z−1=−1\dfrac{2z+1}{z-1}=-1z−12z+1=−1
Même principe ; solution z=0z=0z=0
3ème cas :2z+1z−1=i\dfrac{2z+1}{z-1}=iz−12z+1=i
Même principe ; solution z=−15−35iz=-\dfrac{1}{5}-\dfrac{3}{5}iz=−51−53i
4ème cas :2z+1z−1=−i\dfrac{2z+1}{z-1}=-iz−12z+1=−i
Même principe ; solution z=−15+35iz=-\dfrac{1}{5}+\dfrac{3}{5}iz=−51+53i
Un conseil @Joyca2 , pour t'entraîner, tu fais les deux méthodes et tu vérifies que tu trouves pareil.
Bons calculs.
Tu peux faire une représentation graphique des 6 solutions de l'équation de départ.
Les points A,B,C sont les images des solutions de z3=−1z^3=-1z3=−1
Ils sont sur le cercle de centre O et de rayon 1
Ils sont les sommets du triangle équilatéral (rouge)
Les points D,E,F sont les images ds solutions de z3=2iz^3=2iz3=2i
Ils sont sur le cercle de centre O et de rayon 2132^\dfrac{1}{3}231
Ils sont les sommets du triangle équilatéral (bleu)
@Joyca2 , pour information,
Pour faire le graphique joint, j'ai tout simplement utilisé "geogébra" en lui indiquant 2182^\dfrac{1}{8}281 et 3π16\dfrac{3\pi}{16}163π, c'est à dire pour M0 :
les coordonnées (218cos(3π16),218sin(3π16)\biggr(2^\dfrac{1}{8}cos(\dfrac{3\pi}{16}), 2^\dfrac{1}{8}sin(\dfrac{3\pi}{16}\biggr)(281cos(163π),281sin(163π) et le logiciel fait le job !
Sur papier, tu fais au mieux avec règle graduée, compas, rapporteur, mais dans les calculs ne parle que des valeurs exactes ( 2182^\dfrac{1}{8}281 pour le module et mesures en radians pour les arguments).
Bonsoir,
@Mariem-jabloun ,
Si l'on résume ce qui a été dit,
|z|=3 => ∣z′∣≤103|z'|\le \dfrac{10}{3}∣z′∣≤310
Cela veut dire que |z|=3 => ∣z′∣<103|z'|\lt \dfrac{10}{3}∣z′∣<310 OU ∣z′∣=103|z'|=\dfrac{10}{3}∣z′∣=310
La proposition serait VRAIE si : |z|=3 => ∣z′∣=103|z'|=\dfrac{10}{3}∣z′∣=310 (exclusivement)
Tu as déjà un contre -exemple.
Je t'en indique un autre :
Pour z=3iz=3iz=3i, donc ∣z∣=3|z|=3∣z∣=3, si tu fais le calcul, tu trouves ∣z′∣=83|z'|=\dfrac{8}{3}∣z′∣=38 donc dans cet exemple ∣z′∣<103|z'|\lt \dfrac{10}{3}∣z′∣<310
Par contre, si tu prends z=3z=3z=3 donc ∣z∣=3|z|=3∣z∣=3, tu trouves ∣z′∣=103|z'|=\dfrac{10}{3}∣z′∣=310
En bref, la conclusion ∣z′∣=103|z'|=\dfrac{10}{3}∣z′∣=310 n'est pas toujours exacte.
La proposition de l'énoncé est "FAUSSE".
Je ne sais pas si c'est clair...à toi de juger...
Bonjour,
Bien bizarres les notations dans ce topic.
@sali-sghaier parle parfois de iziziz parfois de −iz-iz−iz
@Black-Jack a dû lire RRR au lieu de iRiRiR (ensemble des imaginaires purs)
Le calcul de @Noemi , en prenant iz∈iRiz \in iRiz∈iR , semble plus cohérent.
Les intervenants le diront.
Bonjour,
Autre méthode possible,@Mariem-jabloun , si tu connais la forme trigonométrique/exponentielle des nombres complexes.
∣z∣=1|z|=1∣z∣=1 vu que M est sur le cercle de centre O et de rayon 1,
z=eiθz=e^{i\theta}z=eiθ
zˉ=e−iθ\bar z=e^{-i\theta}zˉ=e−iθ
z′=eiθ(e−iθ−1)eiθ−1z'=\dfrac{e^{i\theta}(e^{-i\theta}-1)}{e^{i\theta}-1}z′=eiθ−1eiθ(e−iθ−1)
En développant et simplifiant :
z′=e0−eiθeiθ−1=1−eiθeiθ−1z'=\dfrac{e^0-e^{i\theta}}{e^{i\theta}-1}=\dfrac{1-e^{i\theta}}{e^{i\theta}-1}z′=eiθ−1e0−eiθ=eiθ−11−eiθ
Tu transformes et tu trouves −1-1−1 d'où la réponse.
@Ceilan ,
Oui , cette équation x2+y2=5x^2+y^2=5x2+y2=5 aide bien, car en l'utilisant avec x2−y2=−3x^2-y^2=-3x2−y2=−3 et en ajoutant membre à membre, on obtient 2x2=22x^2=22x2=2, d'où x2=1x^2=1x2=1 d'où x=....x=....x=.... et l'équation ab=−2ab=-2ab=−2 permet de déduire yyy.
Sans cette égalité des modules, c'est faisable mais on se retrouve avec du 4ème degré...
Bon travail.
@snd , bonsoir,
Je te mets quelques pistes pour démarrer ton calcul
z=a1+jb1xa2+jb2xz=\dfrac{a_1+jb_1x}{a_2+jb_2x}z=a2+jb2xa1+jb1x
tan(θ2)=b2xa2tan(\theta _2)=\dfrac{b_2x}{a_2}tan(θ2)=a2b2x
Tu pourras, dans tes calculs à venir, remplacer xxx par a2tan(θ2)b2\dfrac{a_2tan(\theta_2)}{b_2}b2a2tan(θ2)
Dans zzz, mets a1a_1a1 en facteur au numérateur et a2a_2a2 en facteur au dénominateur.
z=a1(1+jb1xa1)a2(1+jb2xa2)z=\dfrac{a_1(1+j\dfrac{b_1x}{a_1})}{a_2(1+j\dfrac{b_2x}{a_2})}z=a2(1+ja2b2x)a1(1+ja1b1x)
En remplaçant xxx par l'expression indiquée, ça doit donner, sauf erreur,
z=a1(1+jtan(θ2)a2b1a1b2)a2(1+jtan(θ2))z=\dfrac{a_1(1+jtan(\theta_2)\dfrac{a_2b_1}{a_1b_2})}{a_2(1+jtan(\theta_2))}z=a2(1+jtan(θ2))a1(1+jtan(θ2)a1b2a2b1)
Tu remplaces ensuite tan(θ2)\tan(\theta_2)tan(θ2) par l'expression donnée au début de ton énoncé et tu continues.
Bons calculs !
@Baha-Azouz a dit dans Exercise complexe et ensemble de point :
@Noemi merci beaucoup mais a quoi cert l'information lorsque θ décrit (0,2pe) dans l'énoncé
Car j ai pensé qu il y aura une discussion selon les valeur de θ c est pourquoi j ai pense au formule d euler ?
Bonjour,
A chaque valeur de theta (dans [0 ; 2Pi[) correspond un point M différent.
Tous ces points M constituent un ensemble qu'on te demande de trouver (de décrire).
Par exemple (la bonne réponse n'est pas forcément dans celles proposées ci dessous) :
a) Les points M appartiennent à la droite d'équation ... du plan complexe privée du point d'affixe ...
OU
b) Les points M appartiennent au cercle de centre d'affixe ... et de rayon égal à ...
OU
...
Re Bonjour,
J'espère que tu es arrivé à faire les calculs demandés à la première question de la partie A
Je te démarre la seconde question de la partie A
Tu connais je suppose le principe du raisonnement par récurrence.
Initialisation pour n=1
Tu dois prouver que (z1)‾=(zˉ)1\overline{(z^1)}=(\bar z)^1(z1)=(zˉ)1
Soit z=x+iyz=x+iyz=x+iy
z1=z=x+iyz^1=z=x+iyz1=z=x+iy
z1‾=x−iy\overline{z^1}=x-iyz1=x−iy
zˉ=x−iy\bar z=x-iyzˉ=x−iy
(zˉ)1=zˉ=x−iy(\bar z)^1=\bar z=x-iy(zˉ)1=zˉ=x−iy
Tu tires la conclusion
Hérédité
Tu supposes que pour n (n≥1n\ge 1n≥1) on a (zn)‾=(zˉ)n\overline{(z^n)}=(\bar z)^n(zn)=(zˉ)n
Avec cette hypothèse, tu dois démontrer que :
(zn+1)‾=(zˉ)n+1\overline{(z^{n+1})}=(\bar z)^{n+1}(zn+1)=(zˉ)n+1
Pour faire cette démonstration, tu utilises la propriété démontrée à la question 1)
Essaie, et tiens nous au courant si tu n'y arrives pas ou si tu veux une vérification.
@Noemi et @Melody bonjour,
@Melody , pour vérification,
Si tu as trouvé −12+i32-\dfrac{1}{2}+i\dfrac{\sqrt 3}{2}−21+i23 et −12−i32-\dfrac{1}{2}-i\dfrac{\sqrt 3}{2}−21−i23 , tes calculs sont bons.
Re-bonjour @kadforu ,
Exact pour le 2.
Merci de l'avoir vu. Je viens de compléter. (encore un coup de chaud !)
Remarque vu que le sujet t'intéresse : tu pourrais dire aussi que U→.V→=Re(Z.Z′)ˉ\boxed{\overrightarrow{U}.\overrightarrow{V}=Re(Z.\bar{Z')}}U.V=Re(Z.Z′)ˉ.
Cela est parfois utile, mais pas ici.
La seule interprétation pour la question est celle qui a été indiquée.
Cela nous arrive parfois de voir en TS des demandeurs étrangers qui n'ont pas le même programme qu'en France...On s'adapte.
Bonne soirée !
@Mamadou-Saliou
Quelques résultats pour que tu puisses vérifier les tiens.
Après calculs, sauf erreur,
y1=coskπn−isinkπn=cos−kπn+isin−kπn=e−kiπny_1=cos\dfrac{k\pi}{n}-isin\dfrac{k\pi}{n}=cos\dfrac{-k\pi}{n}+isin\dfrac{-k\pi}{n}=e^{\frac{-ki\pi}{n}}y1=cosnkπ−isinnkπ=cosn−kπ+isinn−kπ=en−kiπ
y1=coskπn+isinkπn=ekiπny_1=cos\dfrac{k\pi}{n}+isin\dfrac{k\pi}{n}=e^{\frac{ki\pi}{n}}y1=cosnkπ+isinnkπ=enkiπ
La valeur S finale, après simplification de produits (il y a beaucoup de travail pour cela !) doit être :
S=1−z2n1−z2S=\dfrac{1-z^{2n}}{1-z^2}S=1−z21−z2n
Bon travail.
@EL
Je suis de plus en plus perplexe sur cet énoncé.
Tu pourrais peut-être t'assurer qu'il est valable...
Vérifie si j'ai bien lu...
Pour la 4) m=1+i
A a pour affixe 1+mi=1+(1+i)i=1+i+i²=1+i-1=i
B a pour affixe 1-mi=1-(1+i)i=1-i-i²=1-i+1=2-i
OA=|i|=1
OB=|2-i|=22+(−1)2=5\sqrt{2^2+(-1)^2}=\sqrt 522+(−1)2=5
Une rotation est une isométrie (conservation des distances)
Or 1≠51\ne \sqrt 51=5
Il n'y a ps de rotation r de centre 0 transformant A en B
Je me pose des questions sur la valeur de cet énoncé ! ! !
Bonjour saraSBH et Noemi,
Noemi, tu as bien fait de compléter ta réponse avec ±\pm±, sinon il aurait manqué une racine carrée complexe.
Je reste plus que perplexe sur l'écriture z\sqrt zz pour z complexe...
saraSBH, dans une précédente discussion sur les racines carrées d'un nombre complexe, je t'ai mis un lien sur le sujet :
https://homeomath2.imingo.net/complex9.htm
Il es précisé clairement :
Alors,pour éviter toute confusion, mieux vaut écrire en français : "les racines carrées complexes de Z sont ..."
Tu peux demander son avis à ton professeur de mathématiques.
Bon travail avec les complexes !
Bonjour Noemi, Ban,
Je confirme , Noemi, il n'y a pas d'erreur, 4zz′=−2+103i4zz'=-2+10\sqrt 3 i4zz′=−2+103i
Je détaille un peu si besoin vu que Ban semble avoir des problèmes sur la "règle de distributivité" qui est la même pour les complexes que pour les réels.
4z′=−1+23i4z'=-1+2\sqrt 3i4z′=−1+23i
4zz′=z(4z′)=(4−3i)(−2+23i)4zz'=z(4z')=(4-\sqrt 3 i)(-2+2\sqrt 3 i)4zz′=z(4z′)=(4−3i)(−2+23i)
4zz′=4(−2)+4(23i)−3i(−2)−3i(23i)4zz'=4(-2)+4(2\sqrt 3 i)-\sqrt 3 i(-2)-\sqrt 3i(2\sqrt 3i)4zz′=4(−2)+4(23i)−3i(−2)−3i(23i)
Après calculs, on trouve la réponse indiquée.
Je trouve que ces valeurs z, z' ne sont guère pertinentes car il n'y a aucune astuce pour le calcul.. en bref que du calcul...
Bonjour,
Ta méthode ne me parait pas très simple...
Quelques idées éventuelles à creuser
Tout d'abord, tu peux prouver facilement (avec les arguments) les équivalences
A , B , 0 alignés <=> ab\frac{a}{b} ba réel
A , B , 0 alignés <=> ba\frac{b}{a} ab réel
C'es équivalences pourront être utilisées pour la partie directe et la partie réciproque.
Soit A , B O alignés
(a+b)2ab=a2+b2+2abab\frac{(a+b)^2 }{ab}=\frac{a^2+b^2+2ab}{ab}ab(a+b)2=aba2+b2+2ab
En décomposant
(a+b)2ab=a2ab+b2ab+2abab\frac{(a+b)^2 }{ab}=\frac{a^2}{ab}+\frac{b^2}{ab}+\frac{2ab}{ab}ab(a+b)2=aba2+abb2+ab2ab
En simplifiant
(a+b)2ab=ab+ba+2\frac{(a+b)^2 }{ab}=\frac{a}{b}+\frac{b}{a}+2 ab(a+b)2=ba+ab+2
ab∈R\frac{a}{b} \in Rba∈R et ba∈R\frac{b}{a} \in Rab∈R et 2∈R2\in R2∈R donc (a+b)2ab∈R\frac{(a+b)^2 }{ab}\in Rab(a+b)2∈R
CQFD
Piste pour la réciproque
Soit (a+b)2ab∈R\frac{(a+b)^2 }{ab}\in Rab(a+b)2∈R
Avec la décomposition précédente
ab+ba+2∈R\frac{a}{b}+\frac{b}{a}+2 \in Rba+ab+2∈R
2 étant réel , tu déduis ab+ba∈R\frac{a}{b}+\frac{b}{a} \in Rba+ab∈R
Tu peux raisonner par l'absurde pour terminer en prouvant que si ab\frac{a}{b}ba n'est pas réel, son inverse ba\frac{b}{a}ab n'est pas réel donc que leur somme ne peut pas être réelle
conclusion :
nécessairement ab\frac{a}{b}ba est réel donc A , B, O sont alignés.
CQFD
Sympa mathix de répondre à une question posée en septembre 2007 !!!! le poseur de question doit avoir fini par avoir le bac depuis un certain temps, il doit même avoir fini ses études !
Bonjour. J'ai lu tout cette discussion ( datant de 2006 quand meme.). Et j'ai le meme exercice en DM et je bloque a la question exprimer OMn en fonction de n.
J'ai trouver:
OMn = |Zn| = | Z0Z_0Z0 <em>(i/2)n<em>(i/2)^n<em>(i/2)n =2exp i</em>∏/3^{i</em> ∏/3}i</em>∏/3 *(i/2)n.
Je sais que la réponse est 2∗(1/2)n2*(1/2)^n2∗(1/2)n mais je n'arrive pas a trouver le calcul. Je ne sais pas comment multiplier une exponentielle avec une puissance...
Merci.
donc l'ensemble E ⇔ à l'équation z = 1 + |z+1|e iθ^{iθ}iθ
et l'ensemble F, correspond a une droite verticale d'équation x=-1, car (z+1) est un imaginaire pure ?
niki112
Bonjour tout le monde!
J'ai une exercice a faire, du type question ouverte (un truc que je deteste profondement!!)
Bref, voici l'enoncé:
Quels sont les nombres complexes z tels z, 1/z et 1+z aient le même module?
Ce que j'ai fait pour le moment:
j'ai posé |z| = |1/z| = 1/|z|
puis |z| = 1 la distance OM = 1 (l'histoire des ensembles pour tout point M...)
et apres si je marque
|z| = |1+z|
OM = AM où A(1)
je peux dire que M appartient à la mediatrice de [OA]
et finalement M1 et M2 sont les intersections du cercle de centre O et de rayon 1 et de la la mediatrice de [OA]
Affixe de M1 z1 = 1/2 + i √(3/2)
Affixe de M2 z2 = 1/2 - i √(3/2)
En fait je sais pas si je me suis embrouillée dans la redaction quelque part...
merci d'avance pour une courte verification
bye
Bonjour ,
Il me semble déceler une erreur : ce n'est pas OM = AM mais OM = OP où P est le point d'affixe 1 + z ( OAPM parallélogramme ) .
Si θ est l'argument de z , on doit avoir ( sauf erreur dans mes calculs ) :
cos θ = - 1/2 et non pas 1/2 .
Merci de vérifier .
Bon courage .
C’est vrai . . . rien n’est plus simple que les maths . . . une fois la réponse sous les yeux. (rire jaune)*****
***** Smilie déposé par Zorro, bien plus chic qu’un vulgaire .
C’est juste un petit clin d’œil sympathique . . . pas me virer (peur bleue)
Bonjour,
Réponse tardive . . .
On veut que z soit imaginaire, pour cela, il faut et il suffit qu’il ne soit pas réel.
Il me semble que tu confonds imaginaire et imaginaire pur.
Rien ne vaut quelques exemples :
Z1 [ [z| ; 3pi ] est réel
Z2 [ [z| ; 2pi/3 ] est imaginaire (et non réel)
Z3 [ [z| ; -pi/2 ] est imaginaire pur
C’est mieux comme ça ?
Bonjour,
Avec les règles de calcul sur les modules et arguments rappelées par Thierry, tu fais :
z1 = z^2 / z’^3
avec z= 1+i et z’=1-i dont les mod et arg sont faciles à trouver
ex avec z : |z| = √2
donc z = √2 x [1/√2 + i (1/√2)] = √2 [cos (pi/4) + i sin (pi/4)]
arg(z) = pi/4
Tu fais la même chose pour 1-i pour trouver mod(z’)=√2 et arg(z’)=-pi/4
Il reste à calculer :
|z1| = |z^2 / z’^3| = |z^2| / |z’^3| = |z|^2 / |z’|^3 = . . .
arg(z1) = arg(z^2 / z’^3) = arg(z^2) – arg (z’^3) = 2 x arg(z) – 3 x
arg (z’) = . . .
Ca devrait aller maintenant je pense. . .
Conseil pioché dans ‘’Prépabac’’ : N’utiliser la forme algébrique que lorsque l’on ne peut faire autrement
Bon, j'ai du boulot
2a) Exprime d'une part zn+2z_{n+2}zn+2 en fonction de zn+1z_{n+1}zn+1 et d'autre part zn+1z_{n+1}zn+1 en fonction de znz_nzn. Déduis-en Δn+1_{n+1}n+1 en fonction de Δn_nn.
2b) Exprime Δ<em>n<em>n<em>n en fonction de Δ</em>n−1</em>{n-1}</em>n−1 grâce à 2a) puis, dans l'expression obtenue, Δ<em>n−1<em>{n-1}<em>n−1 en fonction de Δ</em>n−2</em>{n-2}</em>n−2. Répète l'opération jusqu'à avoir Δn_nn en fonction de Δ0_00.
2c) Utilise la fonction log (log(n)=ln(n)/ln(10)) qui est strictement croissante (donc préserve les inégalités strictes) et qui vérifie log(xnlog(x^nlog(xn)=n*log(x). Enfin la fonction partie entière peut t'être utile si tu n'as pas de calculatrice sous la main pour exprimer n0n_0n0.
J'espère t'avoir aidé...
un grand merci, c'est une partie du cours, je pense que vous l'avez remarquez, que je n'avais pas comprise, on avait fais une unique exemple avec un norme de 1 je pensais que c'était toujours avec 1 qu'il fallait raisonner... Merci beaucoup ça me permettras peut-être de finir l'exercice
Bonsoir.
Malgré les erreurs dans ton énoncé et sachant qu'on ne peut répondre à rien si on ne connais pas la question (on a pas ton DM sous les yeux) je pense qu'on t'a déjà donné pas mal d'indices.
La relation que tu dois trouver, je te l'ai donné. Du moins pour le sens direct, il faut que tu montres que si M' est l'image d'un point du plan par la transformation que tu as donné, alors |z'|≤1.
Vu que la réciproque est vraie (parait-il) il est donc probable que ton énoncé te demande de la démontrer. Pour ça tu dois montrer que si un un complexe z' est tel que |z'|≤1 alors le point M' d'affixe z' est l'image d'un point M du plan.
Dit comme ça, ça semble long à faire mais en fait il y a de bonnes chances que le tout se démontre par équivalences et donc que tu démontre le sens direct et la réciproque en même temps.
Jeet-Chris dit qu'il faut passer à la notation exponentielle. C'est aussi ce que j'aurais fait. Ça à l'air d'être une bonne piste à suivre.
Tu demande quelle relation tu dois trouver alors qu'on te l'a déjà dit et qu'on t'a même dit comment y parvenir. A toi de faire un petit effort. On ne répondra pas à ta place.
Salut à tous !
J'ai un problème sur ce DM. Après avoir bataillé des heures dessus je sollicite votre aide pour résoudre cette énigme :
"Si l'on permute les 2 aiguilles d'une montre, on obtient en général une position impossible sur une montre normale.
Par exemple, s'il est 4h00, la petite aiguille est sur le 4, la grande sur le 0. La permutation verrrait donc la petite aiguille sur le 0 et la grande sur le 4, ce qui est impossible. En effet, s'il est 0h20, la petite aiguille aurait du parcourir 1/3 de l'angle correspondant à 5mn soit 1/3 de 360°/12 soit 10° d'inclinaison par rapport à la verticale.
Question : lors d'un tour d'horloge complet, combien de permutations sont-elles possibles ?
Indications : 1. z^n - (e^(i))^n = ^n e^(in) : l'argument a été multiplié par 2
2. On pourra utiliser des nombres de module 1 représentant les 2 aiguilles et considérer que la grande aiguille tourne 12 fois plus vite que les autres."
Merci d'avance pour votre aide !
Saha
il suffit d'appliquer la définition, en écrivant déjà x+iy-3i = x + (y-3)i. tu vois que le conjugué dont tu as besoin est
x
-(y-3)i = x-iy+3i
cela revient à changer le signe devant chaque imaginaire pur.
Salut.
a=22eiπ4=22(cos(π4)+isin(π4))a=\frac{\sqrt{2}}{2} e^{i\frac{\pi}{4}}=\frac{\sqrt{2}}{2}\left( \cos(\frac{\pi}{4})+i\sin(\frac{\pi}{4})\right)a=22ei4π=22(cos(4π)+isin(4π))
Module : tu n'as pas le droit de faire comme ça. En général |a-1|≠|a|-|1|.
Par exemple si a=-5, alors |a-1|=6, alors que |a|-|1|=4.
En revanche tu peux partir de la deuxième forme de a que j'ai écrite plus haut. Après avoir remplacé le cosinus et le sinus par leur valeur, tu vas pouvoir te ramener à une expression simple de a-1, puis calculer son module.
Formes algébriques des ZnZ_nZn : où est passée l'exponentielle ?
A partir de là je comprends ton problème. Tu as oublié de prendre en compte l'argument de a dans son expression. Ce qui explique ton calcul du module de a-1 qui s'avère tout à fait correct alors.
Pars de mon expression de a et réessaie de faire l'exercice ; ce devrait aller mieux.
@+
bonjours a tous !!
je suis bloqué sur un DM de maths, voila ce que j'ai trouver :
Exo nombre complexe
Le plan est rapporté a un repère orthonormal direct (O,u,v)
On appelle f l'application qui a tout point M d'affixe z ( z≠1) associe le point M' d'affixe z' telle que : z' = (-iz-2) / (z+1)
Soient A,B et C les points d'affixe respectives a= -1, b= 2i et c=-i
Soit C' l'image du point C par f. Donner l'affixe c' du point C' sous forme algébrique puis sous forme trigonométrique.
z' = (-iz-2) / (z+1)
on remplace z par -i donc z' = (-i * -i -2) / (-i +1) = (-3)/(1-i)
l'affixe est donc c' = ((-3) / (1-i)) * ((1+i) / (1+i)) = (-3/2)-(3/2)i
et sous forme trigo: |c'| = √((-3/2)² + ( -3/2)²) = (3√2) / 2
cos θ = -(√2/2)
sin θ= -(√2/2)
d'apres le cercle trigo nous avons 5π / 4
Calculer l'affixe d du point D ayant pour image par f le point D' d'affixe d'=1/2.
on utilise la relation (-iz-2)/(z+1) = 1/2
je passe tout les calculs et je trouve z = 1-2i
Pour tout nombre complexe z différent de -1 on note p le module de z+1 ( c'est a dire |z+1| = p) et p' le module de z'+i(c'est a dire |z'+i|=p')
a)Démontrer que, pour tout nombre complexe z différent de -1,on a : pp' = √5.
j'ai la relation qui est est(enfin je pense)
|z+1|*|z'+i|=pp'=√5
mais je n'arrive pas a la calculer
b)Si le point M appartient au cercle (T) de centre A de rayon 2, montrer alors que M'=f(M) appartient au cercle (T') dont on précisera le centre et le rayon.
je ne comprend pas comment faire ici
Exo trigo
Soit f la fonction définie sur R par : f(x) = sin(x)(cos(x)+1)
Justifier que l'on peut réduire l'intervalle d'étude à [0;π].
sin et cos sont 2π périodique, sin est impaire et cos est paire donc on peut dire que f=2π périodique d'intervalle [-π ; π] mais impaire donc [0;π]
2.Montrer que f'(x) = (cos(x)+1)(2cos(x)-1) puis dresser le tableau de variations de f sur [0;π] (on justifiera les signes trouvés dans le tableau).
je n'arrive pas a calculer cette dérivé ( petit probleme au niveau trigonométrique)
ensuite, on peut dire que (cos x +1) est toujour positif car cos > -1
(2cos x -1) change de signe a π/3 car
2cos x -1>0
cos x>1/2
donc cos x >π/3
le tableau :
x__0_____π/3_____π
f'____+0-_
f____croit____decroit
3.On appelle T la tangente à la courbe Cf au point d'abscisse 0.Déterminer une équation de T.
pas compris
Construire T et Cf sur l'intervalle [-π;3π] en justifiant la construction.
compris mais pas encore fait
je vous remercie d'avance, ce que je n'ai pas trouver est en bleu
Que valent AM et BM ? Tu peux les calculer comme modules de am⃗\vec{am}am et bm⃗\vec{bm}bm ... Il faudra ensuite, pour chacun, trouver par qui le multiplier pour obtenir ce qu'on te demande.
Salut Arwelia,
*méthode algébrique : je n'ai pas vérifier que Y est juste mais s'il l'est, tu as ce système d'équation : y²-4x+4y+x²=0 et (-4+x)²+y² ≠ 0.
Pour la première équation, tu fais bien de passer à la forme canonique, mais tu as fait une erreur de signe, ce n'est pas -8 mais 8... ce qui est plus cohérent pour le carré d'un rayon ! ensuite tu peux voir y+2 comme y-(-2)... Tu as alors tout ce qu'il faut pour reconnaitre le cercle.
Quant à la deuxième équation elle est équivalente à y≠0 et x≠4, à toi de voir si le point (4,0) appartient au cercle précédent...
*méthode géométrique :
si un nombre est réél, ce nombre multiplié par i est imaginaire pur. Et si un nombre est imaginaire pur, ce nombre multiplié par i est réél...
Citation
Pour AM (z-zA)
Pour BM (z-4)
Euh... non, vérifie ce que tu as écris...
3) on cherche l'ensemble des points tels que Z soit réél, donc tel que z+4iz−4\frac{z+4i}{z-4}z−4z+4i soit imaginaire pur, si ce nombre est un imaginaire pur, que vaut son argument ?
Bonjour!
Désolée de revenir vous embeter mais voilà j'ai encore un problème avec la suite...
Pour la partie que je vous ai notée plus haut, c'est ok, l'ensemble C cherché est le cercle qui a pour centre le point d'affixe -1/2 et pour rayon ½.
Mais pour la suite, j'ai du mal...
M est un point de E d'affixe z, r est le module de z et a est l'argument de z, a est compris entre -Pi et Pi.
Je dois démontrer que l'ensemble F des points M de E, tels que l'affixe de P soit un réel strictement positif, est la réunion de trois demi-droites (eventuellement privées de points).
J'ai déjà mis le temps à comprendre la question, et à comprendre qu'on nous disait ce qu'on devait trouver (l'ensemble est constitué de trois demi-droites).
Bon alors tout d'abord je me suis dit que vu que l'affixe de P (z³) doit être un réel, cela signifie que Im(z³)=0. (Je ne sais pas si vous le notez comme ça mais ça veut dire: partie immaginaire de z³ égale 0).
Alors j'ai noté z=x+iy avec x et y des réels.
Puis j'ai cherché la forme algébrique de z³:
z³=(x+iy)³ et j'ai trouvé: z³=[x(x²-3y²)]¨+[iy(3x²-y²)]
Mais comme z³ est un réel, alors y(3x²-y²)=0
donc z³=x(x²-3y²)
Or il faut que ce soit un réel strictement positif donc:
z³>0
x(x²-3y²)>0
donc soit: x>0
soit x²-3y²>0
Mais là, ça veut dire quoi?? que x²>3y²?
Donc que x>√(3)y ?? ou que y<x/√(3) ???
La je bloque
Si quelqu'un pouvait m'apporter son aide^^ Et merci déjà à tout ceux qui auront eu le courage de lire ce problème !!
Bonjour,
Tu es nouveau et tu étais tellement pressé que tu as pas eu le temps de lire : la FAQ = Foire Aux Questions en cliquant ici
ainsi que le message écrit en rouge dans la page d'accueil : Poster son 1er message ici
Tu ne sais donc pas qu'il ne faut poster qu'un énoncé par fil de discussion et qu'il faut nous indiquer ce que tu a déjà cherché et trouvé ou non (et dans ce cas pourquoi tu n'as pas trouvé)
Ici, tu es sur un forum d'aide, pas sur un site qui distribue les réponses !
Je verrouille ce sujet, le temps de faire un copier-coller dans 2 nouveaux sujets qui respectent les consignes à suivre ici et que tu auras lu !
Bonjour,
j''ai quelques questions d'un devoir auxquelles je n'arrive pas a repondre
quelles sont les solutions dans C de l'équation (2z+1/z−1)4(2z+1/z-1)^4(2z+1/z−1)4
auparavant j'ai trouvé les solutions de P(z)=0 ( avec P(z)= z4z^4z4-1,
S={-i;i;-1;1}
mais je n'arrive pas a trouvé les solutions de (2z+1/z−1)4(2z+1/z-1)^4(2z+1/z−1)4.
( qui n'a rien a voir avec la question précédente)
démontrer que Vn est une suite géométrique en sachant que Vn=n(3-Un) et que (Un) est définie sur N* par U1U_1U1= 1 et Un+1U_{n+1}Un+1= (n/2(n+1))Un(n/2(n+1))U_n(n/2(n+1))Un+ (3(n+2)/2(n+1))
j'ai voulu calculer Vn+1V_{n+1}Vn+1 pour trouver Vn+1V_{n+1}Vn+1= q * (n(3−Un(n(3-U_n(n(3−Un) mais je n'y arrive pas ....
( qui n'a encore rien a voir)
déterminer les limites en +∞ et -∞ de f(x)= x/(1+x+x²)
je trouve un cas d'indétermination du type "∞/∞" et je n'arrive pas a trouver la limite en changeant l'expression...
... je sais qu'il y a beaucoup de questions :s
Merci Beaucoup
swann
salut
Citation
b) Exprimer les modules et argument de z' en fonction de ceux de z.
En déduire que O, M et M' sont alignés.
avec les arguments !
deux choses peuvent se produire : 1° si tu as z = k z' avec k∈R, alors z et z' font un angle nul ou plat (0 ou pipipi) donc O, M et M' sont alignés.
2° tu as z = c z', où c∈C*R* alors il y a (entre autre) une rotation entre OM et OM' dont l'angle est donné par c.
Bonjour à tous, pourriez vous m'aider s'il vous plait sur cet exercice avec lequel j'ai beaucoup de difficultés alors que j'ai essayé de le faire plusieurs fois. merci d'avance.
Exercice 4
Dans cet exercice on dispose d'un repère orthonormal direct (o;vecteur u ;vecteur v). On note M'(z') l'image d'un point M(z) par la transformation indiquée.
A,B et c sont les images des complexes zA=3+4i, zB= 7+i et zC=4+2i
Pour tout nombre complexe z on définit Z'=((3+4i)/5)Z'
a) Calculer les affixes de a', B' et C'. Dans un repère, placer A, B, C, A', b', C'.
b) On cherche les points invariants par cette transformation.
Montrer que M'=M ⇔M appartient à une droite D.
Donner l'equation de la droite D et tracer cette droite.
c) Montrer que pour tout nombre complexe z: (z')'=z
d) Exprimer le module de z' en fonction de celui de z.
e) Exprimer l'argument de z' en fonction de celui de z. (on notera téta l'argument de 3+4i).
Montrer que la moyenne des arguments de z et z' est constante.
Quelle propriété géométrique correspond à ce résultat.
f)Quelle semble être la transformation géométrique correspondant à z→z' ?
Bonjour à tous, pourriez vous m'aider s'il vous plait sur cet exercice avec lequel j'ai beaucoup de difficultés alors que j'ai essayé de le faire plusieurs fois. merci d'avance.
Exercice 6
z0=2 et on définit une suite de nombres complexes telle que, pour tout n de , zn+1z_{n+1}zn+1=(1+i)zn.
Pour tout n de l'ensemble N, Mn est le point d'affixe znz_nzn.
a) Calculer z1, z2, z3, z4. Placer les points A1,A2,A3,A4.
b)) Montrer que, pour tout n de l'ensemble N, znz_nzn= 2(1+i)n2(1+i)^n2(1+i)n.
c) Detreminer l'argument de zn pour tout n de l'ensemble N .
d) Exprimer, pour tout n de l'ensemble N, module et argument de zn+1z_{n+1}zn+1 en fonction de ceux de zn.
e) En déduire que l'on passe du point MnM_nMn au point Mn+1M_{n+1}Mn+1 en appliquant successivement une rotation et une homothétie que l'on précisera (centre, angle, rapport).
Bonjour,
Ton énoncé n'est pas vraiment facile à suivre sur ton image !
As-tu réussi à placer K ? Pourrais-tu coder, sur ton dessin, les informations données par l'énoncé ?
Si tu ne reçois pas de réponses, c'est qu'on ne voit pas très bien ce que tu as déjà cherché et trouvé et ce que tu as déjà cherché et pas trouvé !
ET si tu refais un dessin , évite de nous poster un timbre-post pour image !
Bonjour à tous
Le plan complexe est muni d'un repère orthonormal (O;vectU , vectV) direct. Soit A le point d'affixe i et B le point d'affixe -i.
Soit f la fonction définie sur C{i} par f(z)=(1-iz) / (z-i)
Vérifier que pour tout z de C{i}, f(z)=-i + 2/(z-i)
2)a- Démontrer que -i n'a pas d'antécédent par f
b-Déterminer les antécédents de 0 et de i par f
3)A tout point M différent de A, d'affixe z, on associe le point M' d'affixe z' tel que z' = f(z)
a-Démontrer que pour tout point M différent de A, le produit des longueurs AM et BM' est égal à 2.
b-Démontrer que lorsque M décrit le cercle C de centre A et de rayon 4 M' se déplace sur un cercle C' dont on précisera le centre et le rayon
4)a-Déterminer l'ensemble C des points M(z) tels que z'-i soit un nombre réel non nul
b-Démontrer que lorsque M d'écrit C, M' se déplace sur une droite que l'on précisera.
c-Lorsque M décrit C, M' décrit-il toute la droite
5)Déterminer l'ensemble des points M(z) tels que f(z) soit un imaginaire pur non nul.
Réponses:
f(z)= -i + 2/(z-i)
f(z)= -i(z-i) +2/(z-i)
f(z)= (1-iz) / (z-i)
2)a) S'il existe une solution à f(z) = -i, alors cette solution satisfait à :
-i + 2/(z-i) = -i
2/(z-i) = 0
Or il n'existe aucune valeur de z telle que 1/(z-i) = 0
Donc -i n'a pas d'antécédent.
b) f(z)=0
z = -i = 1/i
f(z)=i
z=0
Mais pour la suite je ne trouve pas pouvez vous m'aidez svp ? merci
Ne serait-ce pas le point A qui a pour image le point C d'affixe 1+i ? Dans ce cas pourquoi? L'énoncé me semble peu clair et je n'ai même pas compris pourquoi dans la question 1)a. on utilise l'affixe z' en faisant z'o'. Quelqun me l'a fait vite fait mais je n'ai pas compris. avant de poster cet exercice je voulais montrer que j'avais cherché mais je n'arrive à rien avec cet exercice je n'arrivais même pas à faire la première question. J'ai un niveau très faible en math et je suis un peu découragée de plus cet exercice semble d'un niveau assez élevé et c'est un dm de math que je dois rendre pour la rentrée.
Je vous remercie pour votre aide.
Juste une petite remarque en passant sur ce pauvre M. De Moivre dont le nom est écorché à chaque utilisation de sa formule puisqu'il s'agit en fait de la formule de De Moivre... C'est certes un poil plus lourd à dire (et à écrire) mais respectons ce mathématicien !
Je rappelle également que De Morgan a le droit a sa particule...
Je dois me déconnecter et demain je pense que j'aurai peu de temps pour le forum, mais il y aura bien une bonne âme qui passera par là.
Bon travail et n'oublie pas ce que tu as appris les années précedentes !
Re, oui Df=]0,+inf[ mea culpa!
Maintenant , l'exercice:
a) Formule d'une rotation de centre C et d'angle Θ
(Merci Wikipedia!)
et exp(iπ/2)=i (π=pi)
d'ou
z'=iz
donc x'+iy'=i(x+iy)
x'+iy'=-y+ix
Or deux complexes sont égaux si et seulement si leurs parties réelles et imaginaires sont égales deux à deux.
donc, on a x'=-y et y'=x
(C'est bon!)
b)
Tes résultats sont bons A' (0, 3), B' (ln2, 5/4), P' (0, 5/4) !
par pure déduction de la question a).
2)Soit M(z) un point du plan, M∈C et soit un point M'(z') l'image de M(z) par r(0,π/2). (On admet que lorsque le point M décrit C, le point M' décrit C'.)
On sait que:
z'=iz
d'ou x'=-y et y'=x
Or M∈C donc la partie réelle de z reste x mais la partie imaginaire est f(x)
soit y=ln [√(1+x) - 1 ]
d'ou
x'=-ln[√(1+y')-1]
exp(-x')=√(1+y')-1
(exp(-x')+1)²=1+y'
y'=(exp(-x)1)(exp(-x)+1+1)
y'=exp(-x)(exp(-x)+2)
y'=exp(-2x)+2exp(-x)=g(x)
d'ou z'=x'+ig(x)
donc M'(z')∈C'
Voila, j'espere t'avoir aidé!
Reviens si quelque chose ne va pas!
Salut pacman,
Pourrais-tu écrire les résultats que tu as déjà obtenus, cela nous éviterait de refaire tout ton problème.
Pour la 2-b un nombre complexe est réel lorsque son argument est congru à 0 modulo π, à toi de voir quand est-ce que c'est le cas...
Salut
Citation
je pose la question parce que ces exercices sont des applications censées être pour les equation de 1er degré et là je vois du second degré apparaitre....
*Ce sont des équations du second degré mais elle ne font qu'intervenir la méthode de résolution du 1er degré.
b) on pose z=x+iy
alors z(barre)=x-iy
|x-iy+1/2|=4 => √(x+1/2)²+(y-0)²=4
Tu devrais reconnaitre l'équation d'un cercle de centre (-1/2,0) de rayon 4
Bon courage
je vois..
et est-ce que je peux rajouter:
(zn+1 - zn) / zn+1 = i√(3)
zn+1 - zn = zn+1i√(3)
|zn+1 - zn| = |zn+1i√(3)|
|zn+1 - zn| = |i√(3)| × |zn+1|
|zn+1 - zn| = √(3) |zn+1|
sachant que nulle part dans mon cours c'est explicitement écrit que |z×z'| = |z| × |z'|?
Lol.
Merci.ça me fait plaisir que tu aies trouvé z=1.
J'tavoue que pendant 25 minutes j'ai fait pas mal de systèmes se ramenant tous a Z1= 7+i/3 ,ce qui ne m'arrangeait pas vraiment ^^
Donc Merci beaucoup pour cette aide
Bonjour,
Comme tu l'as dit :
MMMnM</em>n+1M</em>{n+1}M</em>n+1 = |zn+1z_{n+1}zn+1 - znz_nzn|
en remplaçant ...
|zn+1z_{n+1}zn+1 - znz_nzn|
= |(1/2i)n+1(1/2i)^{n+1}(1/2i)n+1(1+i√3) - (1/2i)n(1/2i)^n(1/2i)n(1+i√3)|
= |(1+i√3)| × | (1/2i)n(1/2i)^n(1/2i)n( (1/2i) - 1 )|
= 2 × |(-i/2)|n^nn × |( 1/2i) - 1)|
Bon je t'ai fait la grosse parti du travail , en fait ce qu'il fallait bien faire c'était extraire la puissance n du module , en effet tu sais d'apres ton cours que |znz^nzn| = |z|n^nn
Bonjour et bienvenue sara94,
Pour savoir comment fonctionne le forum, il y a un message écrit en rouge dans la page d'accueil : Poster son 1er message ici
Je te demande de me donner ,(zˉ)2,,, (\bar {z})^2,,,(zˉ)2,, en fonction de ,z2,, z^2,,z2, pas de ,zˉ,,, \bar {z},,,zˉ,,
Tu vas trouver ,(zˉ)4,,, (\bar {z})^4,,,(zˉ)4,, en fonction de ,z4,, z^4,,z4, pas de ,zˉ,,, \bar {z},,,zˉ,,
On ne va pas reprendre, depuis la 3ème, les cours essentiels absolument nécessaires pour suivre en terminale !
Tu n'as jamais entendu parlé des équations produit ?
A et B étant 2 réels on a : a,×,b,=,0,⟷,a,=,0,,ou,a,=,0,a, \times , b ,=, 0 , \longleftrightarrow , a,=, 0, , \text{ou} , a,=, 0,a,×,b,=,0,⟷,a,=,0,,ou,a,=,0,
Pourrais-tu aussi abandonner une manie que tu as en commun avec surfeuz de citer systématiquement la dernière réponse que tu as eue. On n'est encore pas assez sénile pour croire que tu nous réponds à ce qu'on t'a dit il y a une semaine !
Si tout le monde faisait cela il faudrait un serveur encore plus volumineux pour enregistrer tous les messages et puis cela ne sert à rien puisque tout le monde doit être comme moi et ne relit pas les citations qu'on a déjà lues ....
coucou
il y a des problèmes de notations
b) (n) est la suite arithmétique réelle de premier terme o (o [2/3] et de raison 2pi/3. Exprimer n en fonction de o et de n.
--> n=Uo+n2pi/3 ???
ce ne serait pas plutot un=u0+n2π3u_n = u_0 + n\frac{2\pi}{3}un=u0+n32π
zn=rneinz_n=rne^{in}zn=rnein
n ici c'est la suite (n) que tu as définis plus haut ou alors c'est n du ∀ n ∈ N
salut
nous avons z'=az+b qui est l'écriture d'une similitude directe avec comme nombre complexe a et b l'affixe d'une translation de forme zbz_bzb=x+iy
on peut donc avoir une rotation alors au niveau de a on aura à determiner l'argument et le module de a.dans notre cas on peut avoir une rotation et une translation.
ba oui je suis inspirée par les dieux tout ce que je dis est, par conséquent, forcément vrai ...
lol
nan mais tu regardes à la calculatrice ...
(je viens de casser quelque chose là ?! mdr)
coucou
aaaaaaaa désolée ce n'est pas de ta faute mais j'ai dû retapé tout mon post parce que je "n'étais pas autorisée à poster "
lol
bon (la manière de Zorro est sans doute plus simple)
arg(z′)=(ma⃗;mb⃗)+π2a2πarg(z')=(\vec {ma}; \vec{ mb})+ \frac{\pi}{2} \text{a} 2\piarg(z′)=(ma;mb)+2πa2π
⇔
arg(z′)−π2=(ma⃗;mb⃗)a2πarg(z')- \frac{\pi}{2} =(\vec {ma}; \vec{ mb}) \text{a} 2 \piarg(z′)−2π=(ma;mb)a2π
⇔
arg(z′)−arg(i)=(ma⃗;mb⃗)a2πarg(z')- arg(i) =(\vec {ma}; \vec{ mb}) \text{a} 2 \piarg(z′)−arg(i)=(ma;mb)a2π
⇔
arg(z′i)=(ma⃗;mb⃗)a2πarg(\frac{z'}{i}) =(\vec {ma}; \vec{ mb}) \text{a} 2 \piarg(iz′)=(ma;mb)a2π
en bidouillant ça devrait aller je pense (j'espère :D)
tu as compris ??
arg(z′i)=arg(iz+3z+i×1i)arg(\frac{z'}{i}) = arg (\frac{iz+3}{z+i} \times \frac{1}{i})arg(iz′)=arg(z+iiz+3×i1)
arg(z′i)=arg(iz+3zi−1)arg(\frac{z'}{i}) = arg (\frac{iz+3}{zi-1})arg(iz′)=arg(zi−1iz+3)
on multiplie par i au numérateur et au dénomintaeur
arg(z′i)=arg(3i−z−i−z)arg(\frac{z'}{i}) = arg (\frac{3i-z}{-i-z})arg(iz′)=arg(−i−z3i−z)
arg(z′i)=arg(zb−zmza−zm)arg(\frac{z'}{i}) = arg (\frac{z_b-z_m}{z_a-z_m})arg(iz′)=arg(za−zmzb−zm)
arg(z′i)=(ma⃗;mb⃗)arg(\frac{z'}{i}) = (\vec{ma};\vec{mb})arg(iz′)=(ma;mb)
lol
nan je préfère faire des maths pour mon plaisir... et puis tout le monde ne peut pas être prof
j'ai 18 ans je suis en fac de ... bio XD
de rien pour l'aide
++++
donc on a ça
z′=z2i−zz'=\frac{z^2}{i-z}z′=i−zz2
et bien pour commencer tu remplaçes zzz par x+iyx+iyx+iy
et tu développes...
il te faudra ensuite séparer la partie réèle de la partie imagniaire et tu auras x′x'x′
Pour que ton Z soit imaginaire pur il faut que sa partie réelle soit nulle donc il faut que
X = 0
Or X est une fraction donc il faut appliquer ce que je te disais le 15/10 à 12h17
Donc le dénominateur de X doit être nul ! A toi de modifier son écriture pour arriver à une équation de cercle !
Pense à la méthode que tu as utilisée en 1ère pour la forme canonique
x2x^2x2 + bx = (x + (b/2) )2)^2)2 - b2b^2b2
Le point à exclure est celui dont les coordonnées rendent le dénominateur nul !
ah mais oui je suis vraiment trop nulle !Je m'étais trompé en remplaçant les z par x+iy alors que le module de z est égal à √(x²+y²) .Et j'ai aboutit au même résultat pour les deux ... enfin ! Merci beaucoup de votre aide !
Sinon... avec l'écriture exponentielle
2+3i = a eibe^{ib}eib
2-3i = a e−ibe^{-ib}e−ib (conjugué)
alors
z3z_3z3 = a2002a^{2002}a2002 (e2002ib(e^{2002ib}(e2002ib + e−2002ibe^{-2002ib}e−2002ib) = a2002a^{2002}a2002 2cos(2002b)
avec une formule d'Euler.
Bonsoir à tous,
J'ai un dm à rendre pour jeudi et je dois dire que même si je m'y prend à l'avance, je reste bloqué sur un exercice dont je vous donne l'énoncé et les questions:
Dans le plan complexe, on considère le point A d'affixe 1 et le cercle de diamètre [O;A] ; O est l'origine. Soit M d'affixe m un point quelconque du cercle, distinct de A et O. On trace les carrés directs MAPN et OMKL. Soient k,l,n,p les affixes des points K,L,N,P.
1°) Faire la figure ---> pas de pb à ce niveau là !!
2°) Démontrer que |m - 1/2| = 1/2
3°) Démontrer que l'on a : l = im et p = -im + 1 +i
4°) Démontrer que l'on a : n = (1-i)m +i et k = (1+i)m
5°)Démontrer que le milieu oméga du segment [PL] est fixe (ne dépend pas de la position de M sur le cercle). Justifier que oméga est sur le cercle.
6°) Démontrer que la distance KN est constante. Quelle est la nature du triangle OmégaNK?
7°) Démontrer que le point N est sur un cercle fixe ( indépendant du point M). Donner son centre et son rayon)
Voilà, j'ai tenté de trouver avec les formules de rotation et ce n'est pas expliqué dans les bouquins non plus .....
Merci beaucoup d'avance de m'aider car je suis bloqué ...... merci